Proving a matrix inequality

Here's a proof of the inequality. Start by setting $\epsilon=\lVert A-B\rVert > 0$. I am using the operator norm on $\mathbb{R}^{m\times m}$, so this is the maximum absolute eigenvalue of $A-B$. Exchanging $A$ and $B$ if necessary, there exists a unit vector $e_1$ with $(B-A)e_1=\epsilon e_1$. Diagonalising the bilinear form $(x,y)=x^TAy$ on the orthogonal complement of $e_1$ we extend to an orthonormal basis $(e_1,e_2,\ldots,e_m)$ with respect to which $A$ is $$ A = \left(\begin{array}{ccc}a_1&-u_2&-u_3&-u_4&\cdots\\ -u_2&a_2&0&0&\cdots\\ -u_3&0&a_3&0&\cdots\\ -u_4&0&0&a_4&\\ \vdots&\vdots&\vdots\end{array}\right) $$ Positive semidefiniteness of $A$ gives $\sum_{k=2}^mu_k^2/a_k\le a_1$ (any terms with $a_k=0$ necessarily have $u_k=0$, and I am setting the ratio to zero). This inequality follows from $x^TAx\ge0$ where $x_1=1$ and $x_k=u_k/a_k$ for $k\ge2$.

Now, choose a $\delta_0$ small enough, the precise value to be chosen later. As we have $m$ distinct intervals $(\delta^2,\delta)$ for $\delta=\delta_0^{2^k}$ ($k=0,1,\ldots,m-1$), at least one of these will be disjoint from $\{\lvert u_k/a_k\rvert\colon k=2,\ldots,m\}$. Let $S$ be the set of $k=2,\ldots,m$ with $a_k=0$ or $\lvert u_k\rvert/a_k\ge\delta$, and $S^\prime=\{2,\ldots,m\}\setminus S$, so that $\lvert u_k\rvert/a_k\le\delta^2$ for $k\in S^\prime$. Define $x\in\mathbb{R}^m$ by $x_1=1$ and $$ x_k=\begin{cases} 0,&k\in S,\\ u_k/a_k,&k\in S^\prime. \end{cases} $$ We can compute $$ (Ax)_1=a_1-\sum_{k\in S^\prime}u_k^2/a_k\ge\sum_{k\in S}u_k^2/a_k\ge\delta\sum_{k\in S}\lvert u_k\rvert\ge\delta u $$ where $u=\sqrt{\sum_{k\in S}u_k^2}$. And, $(Ax)_k=-u_k$ for $k\in S$ and $(Ax)_k=0$ for $k\in S^\prime$.

If we define $C\in\mathbb{R}^{m\times m}$ by $C_{11}=B_{11}=A_{11}+\epsilon$ and $C_{ij}=A_{ij}$ otherwise, then $$ \lVert Cx\rVert=\sqrt{((Ax)_1+\epsilon)^2+u^2}. $$ As $(Ax)_1\ge\delta u$, this implies $$ \lVert Cx\rVert\ge\frac{\epsilon\delta}{\sqrt{1+\delta^2}}+\lVert Ax\rVert. $$ Here, I have used the simple fact that the gradient $(d/da)\sqrt{a^2+u^2}\ge\delta/\sqrt{1+\delta^2}$ over $a\ge\delta u$. We also have $\lVert C-B\rVert\le\epsilon$ and, as $\lvert x_k\rvert\le\delta^2$ for $k=2,\ldots,m$, $$ \left\lvert\lVert Cx\rVert-\lVert Bx\rVert\right\rvert \le\lVert(C-B)x\rVert\le\epsilon\delta^2\sqrt{m-1}. $$ Hence, $$ \lVert Bx\rVert-\lVert Ax\rVert\ge\frac{\epsilon\delta}{\sqrt{1+\delta^2}}-\epsilon\delta^2\sqrt{m-1}. $$ So, as we have $\lVert x\rVert\le\sqrt{1+(m-1)\delta^4}$ and $\epsilon=\lVert A-B\rVert$, $$ \sup_{\lVert x\rVert=1}\left\lvert\lVert Ax\rVert-\lVert Bx\rVert\right\rvert\ge\left(\frac{1}{\sqrt{1+\delta^2}}-\delta\sqrt{m-1}\right)\frac{\delta}{\sqrt{1+(m-1)\delta^4}}\lVert A-B\rVert. $$ As $\delta$ is in the range $\delta_0^{2^{m-1}}\le\delta\le\delta_0$, $$ \sup_{\lVert x\rVert=1}\left\lvert\lVert Ax\rVert-\lVert Bx\rVert\right\rvert\ge\left(\frac{1}{\sqrt{1+\delta_0^2}}-\delta_0\sqrt{m-1}\right)\frac{\delta_0^{2^{m-1}}}{\sqrt{1+(m-1)\delta_0^4}}\lVert A-B\rVert. $$ Choosing $\delta_0$ small enough to make the multiplier on the right hand side positive, which can be done independently of $A,B$ (e.g., $\delta_0=1/(2\sqrt{m})$), gives the result.


Here is the result of my attemps at concluding the proof, using the same approach as the one presented in the question. Without loss of generality, we can assume that the eigenvalues of $B$ occur in descending order in the diagonal. From the last equation in the question, we have: \begin{equation*} \begin{aligned} \left|\sqrt{x^T (B+\Delta)^2 x} - \sqrt{x^T B^2 x}\right| &\geq \frac{\left|2 \, x^T B \Delta x + x^T \Delta^2 x\right|}{2(\sqrt {x^T B^2 x} + \sqrt{x^T \Delta^2 x})} \\ &\geq \frac{2 \, x^T B \Delta x + x^T \Delta^2 x}{2(\sqrt {x^T B^2 x} + 1)} =: R_{B,\Delta}(x) \end{aligned} \end{equation*} We show that there exists $c(m)$ such that for any $M,\Delta$ as above, there exists $x \in \mathbb R^m$ such that $R_{B,\Delta}(x) \geq c(m)$. Assume for contradiction that there exist $(B)_{i \in \mathbb N}$ and $(\Delta)_{i\in\mathbb N}$ such that $\sup_{\|x\|=1} R_{B_i,\Delta_i}(x) \to 0$, and let $x_i$ denote an eigenvector corresponding to the eigenvalue $1$ of $\Delta_i$. Passing to a subsequence if necessary, we can assume that $x_i \to x_\infty$. By assumption, \begin{equation*} R_{B_i,\Delta_i}(x_i) = \frac{2 \, x_i^T B_i x_i + 1}{2\sqrt {x_i^T B_i^2 x_i} + 2} \to 0 \Rightarrow \frac{x_i^T B_i x_i}{\sqrt {x_i^T B_i^2 x_i}} \to 0 \Rightarrow x_i^T \left(\frac{B_i}{\|B_i\|}\right) x_i \to 0, \end{equation*} so in particular the first component of $x_\infty$ is equal to 0 (Because $B_i/\|B_i\|$ is a diagonal matrix whose first entry on the diagonal is equal to $1$). Let now $(y_i)_{i\in\mathbb N}$ be the sequence obtained from $(x_i)_{i\in\mathbb N}$ by replacing the first component by 0. By assumption, \begin{equation*} \begin{aligned} R_{B_i,\Delta_i}(y_i) &= \frac{(B_i\,y_i)^T(\Delta_i y_i)}{\|B_i y_i\| + 1}\, + \,\frac{\|\Delta_i y_i\|^2}{2 \|B_i y_i\| + 2}. \\ &= \underbrace{\frac{y_i^TB_i\,y_i}{\|B_i y_i\| + 1}}_{\text{$\geq 0$}}\, + \underbrace{\frac{(B_i\,y_i)^T\,}{\|B_i y_i\| + 1}}_{\text{vector of norm less than 1}}\,\underbrace{(\Delta_i y_i - y_i)}_{\to 0} + \,\underbrace{\frac{\|\Delta_i y_i\|^2}{2 \|B_i y_i\| + 2}}_{\geq 0} \to 0 \end{aligned} \end{equation*} Since $(\Delta_i y_i - y_i) \to 0$ (because the first component of $x_\infty$ is 0), all terms must tend to 0. In particular, $$\frac{\|\Delta_i y_i\|^2}{2 \|B_i y_i\| + 2} \to 0,$$ so $\|B_i y_i\| \to \infty$, because $\|\Delta_i y_i\| \to 1$. In addition, $$\frac{y_i^TB_i\,y_i}{\|B_i y_i\| + 1} \to 0,$$ and since $\|B_i\,y_i\| \to \infty$ this impies that: $$\frac{y_i^T B_i y_i}{\|B_i y_i\|} \to 0,$$ so the second component of $x_\infty$ is 0 too. Continuing this way, we obtain $x_\infty = 0$, which is a contradiction because $x_i \to x_\infty$ and $\|x_i\| = 1$.


I am not yet satisfied with the following approach. So if you found a bug in it, please don't punch me!

It can be shown that if $A,B$ are symmetric matrices then $$\|Ax\|\cdot \|Bx\|\ge\frac 12 x^T(AB+BA)x\label{*}\tag{*}$$ because $\|u\|\, \|v\|\ge u\cdot v$ implies $\|Ax\|\cdot \|Bx\|\ge x^T A^T Bx$ which can be rearranged as above.

The left-side of your inequality can be written as: $$\sup_{\|x\| = 1} \left|\,\|Ax\| - \|Bx\|\,\right|=\sup_{\|x\| = 1}\left\{\|Ax\|^2 + \|Bx\|^2-2\|Ax\|\cdot \|Bx\|\right\}^{1/2}$$ And due to the definition of the matrix norm, for the right-side: $$\begin{align}\|A-B\|&=\sup_{\|x\| = 1}\|(A-B)x\|\\ &=\sup_{\|x\| = 1}\left\{x^T(A-B)^T(A-B)x\right\}^{1/2}\\ &=\sup_{\|x\| = 1}\left\{x^TA^T Ax+x^T B^T Bx-x^T(A^T B+B^TA)x\right\}^{1/2}\\ &=\sup_{\|x\| = 1}\left\{\|Ax\|^2 + \|Bx\|^2-x^T(AB+BA)x\right\}^{1/2}\end{align}$$ Now you can relate the two sides of the inequality using the $\eqref{*}$.